« first day (2730 days earlier)      last day (2308 days later) » 

9:00 PM
@TedShifrin ^
 
So our coordinate system is specified by $y$ positive going down and $x$ positive going right?
 
@TedShifrin no +ve y up.
 
@TedShifrin Hello Ted :D
 
Anyway, the elongation of A should be, acc to the diagram: $x\cos 90^\circ= 0$ :\
 
Sigh... another office hour passes with no students. :(
 
9:03 PM
@Xander: Even when I had 8 office hours a week, I averaged 6-10 an hour.
 
Ted on your lectures you write something like
 
I'm not really upset; it just frustrates me that many of them are going to fail, and they don't have to!
 
are you unable to understand my messy diagram? Should I redraw it?
 
|x-a|< r is a Square
 
And I typically have lots of students; this quarter is just being weird
 
9:05 PM
I did not get that in that context
 
@Ted @XanderHenderson I'll go and just do homework quietly during office hours, even if I have no questions. Dont know why more dont do that
 
Sorry for delay/distraction. My landlord and plumbers are here working on stuff.
 
If anything, I get to know the professors better and am less scared to approach them
 
@Abcd: So what are the masses of $A$, $B$, and $C$?
@Cookie: That's not bad unless there's a mob of students, as I usually had. But some students would try to sit in the corner.
@Kasmir: No, maybe $|x_i-a_i|<r$ for all $i$?
 
That moment when your whole class decides they want another lecture in your office :P
 
9:08 PM
Heya DogAteMy.
No, not lectures. But they were all working on various homeworks, usually.
 
@TedShifrin Yes =p
 
@CookieToast Last quarter, we had three sections of precalculus with about 600 students total
 
at the end of lecture 14
 
@TedShifrin They are rigid supports, not masses...
 
on youtube list
 
9:08 PM
office hours during the last week typically averaged about 100-150 students
 
@Kasmir: $\|\mathbf x-\mathbf a\|<r$ is a ball.
 
THAT SUCKED
 
A supports spring A, B supports spring B, C supports spring C
 
LOL, @Xander. That is absurd.
 
Yes all the Points that are within r of a
 
9:09 PM
indeed; they all went to the primary instructor's office hours, rather than the TAs
 
less than r
 
The spring C is compressed by x.
 
Oh, @Abcd, there aren't masses?
 
I am trying to find the component of x along Spring A and Spring B.
@TedShifrin yes.
 
9:09 PM
If $C$ is compressed an amount $x$, the vector should be going the other way.
 
@TedShifrin can you also tell me a defintion of compact set? like what it is in easy way ?
 
vector x points towards rigid support C.
 
Every open cover has a finite subcover!
 
The definition in my course isn't the one you're learning, @Kasmir. You have the definition.
@Abcd: No, if you're telling me there's compression and $C$ is fixed, then the vector $x$ is pointing inward, isn't it?
 
@TedShifrin What is "compactness" in your course? And what course is it?
 
9:12 PM
Normally, $x$ would represent the displacement of of the spring from its rest length.
 
ooo, springs
 
@Xander: My multivariable math course that's 112 lectures on YouTube. ... I used closed and bounded as the definition. We were doing only $\Bbb R^n$.
@Semiclassic: Maybe you can help out with Abcd too :)
 
tbh I'm not a fan of writing hooke's law as $F_s=-k(x-\ell)$
 
@TedShifrin Ah. That makes sense. Yay for metric spaces!
 
@Abcd what's the question?
 
9:14 PM
@Semiclassical Let me redraw the diagram and post.
 
I think we're trying to figure out the resultant force on the spring to $A$.
 
@TedShifrin No, I just want the component of x along A...
then I'll be able to solve my question :)
 
kk
My preferred spring problem is two objects connected by a spring, each connected by springs to fixed supports, all on one line
 
Here @Semiclassical
I am trying to find the component of $\vec{x}$ along spring A.
 
okay. What's the statement of the problem?
 
9:18 PM
If the particle is pushed slightly against B and released, find the time period of the oscillation..
 
First I have to vectorially add the forces to do that..
Then it's very easy.
 
Okay. I can help in one of two ways---either to help you finish the approach you're doing, or to suggest another approach
 
Is it $x \cos 60^\circ$?
 
gracefully recedes into the fog
 
9:19 PM
@Semiclassical I won't be able to absorb another approach right now. Too sleepy, it's almost 3 am here.
 
oof, fair enough
 
LOL
 
first thing I'd point out is that it really shouldn't matter which of A,B,C you push the particle towards
 
Sup chat
 
Heya Eric.
 
9:20 PM
yeah, that is obvious.
 
@Kasmir: Did you have a specific question I missed?
 
well. so long as the springs are identical
 
@TedShifrin did you ever work out that Bryant proof for the complex frobenius thing
 
yeah, that's given.
 
I hate getting advice cookies instead of fortune cookies
 
9:21 PM
my point is more that you could just as easily draw the picture with B at the top
 
@TedShifrin I got plenty but noticed you were busy helping others so I stopped ><
 
“Seek contentment.” That’s what I did when I bought lunch, you littl shit
5
 
and then it's a bit easier to follow since the motion is vertical rather than diagonal
 
Oh yes. You are right. Done that on my notebook now.
 
@TedShifrin you still here in like couple of hours?
 
9:21 PM
LOL, @MikeM. @EricSilva: No, I got distracted. I got confused about what dimensions were going on. I should go back and work on it later on.
 
mmkay
 
Mmk
I have to do some work about constructing the heat kernel on Riemannian manifolds that I should be doing RN but I'm lazy
 
now, one further point here is that I think you have to make at least one simplifying assumption here
 
@EricSilva: So the key idea was to throw all the $dz^i$ into the differential ideal, so that $\bar\partial$ turns into $d$ mod the ideal. But then there's a question of what dimension the integral manifolds will have.
 
namely, you should assume that all the springs act at the same point O
 
9:23 PM
Yeah, that's fine.
 
@TedShifrin hmm
 
without that, things get complicated without being terribly interesting
 
@CookieToast: How's the Chapter 1 stuff coming/going?
 
ok.
So, the compression of spring B is pretty obvious
 
Maybe that's why "particle" (instead of block) is given in the question.
 
9:24 PM
yeah
 
@Semiclassical yes, then?
 
@EricSilva: Nirenberg's theorem requires that $\mathscr I \cap \overline{\mathscr I} = \emptyset$, which holds here, I believe.
 
if you move the particle towards B by a distance $\Delta x$, the the compression is $\Delta x$. Nothing strange there.
 
yes
and the extension of the two other springs is $\Delta x\cos 30^\circ$?
 
@Ted I've finished every homework problem except the challenge problems. Its frustrating, but I've had a lot of homework this week and I havent had more than an hour or two this week to work on it.
That being said, I'm pretty much going to use your book instead of my classes for the semester wherever possible.
My professor "proved" the vector equation for $\cos\theta$ by rewriting the law of cosines formula today. Our class' textbook is much the same
 
9:26 PM
That's too quick.
 
@TedShifrin mmmmk
 
Being sarcastic? Am I right?
 
@Cookie: You'll find that my course is a more seriously mathematically thorough course. I've taught plenty of engineering-style multi calculus courses, too :)
 
You may be, but I'm not sure off the top of my head, and I'd prefer to do this more carefully
What're the new and old positions of the particle?
For simplicity, let's take the coordinate system to have B at the top i.e. on the positive y axis
 
$(0,0) \to (0, x)$
 
9:28 PM
@Ted I am a bit farther in the videos though. I managed to get through lecture four before school started!
 
Given a locally Euclidean (locally homeomorphic to some Euclidean space) topological subspace $X\subset\mathbb R^n$ and $p\in X$, let $\mathrm{T}_pX$ denote the *tangent set* of $X$ at $p$, namely the set of derivatives of curves in $X$ based at $p$ and differentiable at zero.

Are the following conditions equivalent?

1. $\mathrm{T}_pX$ is a linear subspace of $\mathbb R^n$ of dimension $\dim_pX$.
2. We have the following equality, where the limit is taken in a translated neighborhood in $X$ of $p$. $$\lim_{h\to 0} \frac{\pi_{(\mathrm T_pX)^\perp}(h)}{\|h\|}=0$$ (Here $\pi_{(\mathrm T_pX)^
 
actually, I'd say $(0,0)\to (0,x)$ based on the fact that $B$ is at the top rather than on the right
you can do it the other way, of course
 
@TedShifrin if A,B subsets of R, and A is open, is the intersection of A and the closure of B contained in the closure of A int B ?
 
edited already...
 
kk
Now, what are the positions of A,B,C?
 
9:30 PM
that's what my question is...
$\Delta x \sin 30^\circ$?
 
Glad you're still having fun, @Cookie :)
I dunno, @Kasmir. Is it?
Do you have a way of thinking of the closure other than the definition?
 
hmm
thats the thing Ted, I dont have any geometric thing in my head
 
going to sleep now, good night. Please verify my answer if possible @semi. Bye...
 
so no intuition thats why I asked you :D you allways got a good pic :D
 
@Abcd think about it in the morning
 
9:35 PM
It's causing some serious ethical dilemmas! Do I go to bed like a responsible person, or do I stay up and continue playing with the math? @TedShifrin Luckily I have coffee haha
 
yeah, sure.
 
@TedShifrin My preferred way to do these problems is always to start from the fact that $\vec{F}_{net}=0$ at equillibrium
 
@MikeMiller may I ask for your intuition?
 
@Semiclassic: Sure, me too. That's tacit in how he's thinking.
 
eh. not if you include $\ell$
 
9:36 PM
@Kasmir: Have you guys not discussed limit points?
 
Go for it.
 
The equillibrium length is always a statement about a hypothetical scenario involving only that spring; it has nothing immediately to do with the actual equillibrium which involves all three
 
Do you think the conditions in the above question are equivalent?
 
Funny guy, @Cookie :)
 
What matters it that $\vec{F}_{net}(\vec{x}+\Delta \vec{x})=\vec{F}_{net}+\Delta \vec{F}_{net}=\Delta \vec{F}_{net}$
 
9:39 PM
Oh I have no idea.
 
@TedShifrin the way I Think about it is that p is a limit Point if for any nbhd of p ,it contains Point of our set
 
@MikeMiller not even a hunch? :(
 
That last thing in particular is too wild for me to have an intuition for. But I have essentially never thought about tangent spaces to nasty bois.
 
Point different from p i should add
 
Maybe the Whitney umbrella once in my life, or a Zariski tangent space...
 
9:40 PM
Why is that last thing wild? It's just asserting tangency of the tangent set
 
@Kasmir: How do you see what the closure of $B$ is in terms of limit points?
 
I rarely understand what formulas "mean" at a glance
 
@Arrow: But what does $(T_p X)^\perp$ mean if $T_pX$ isn't a linear subspace?
 
I guess he takes the linear closure
 
@TedShifrin sorry, I should have asked for the mere existence of some linear subspace satisfying this condition. I got ahead of myself and wrote it's necessarily the same creature.
 
9:42 PM
"There is a vector subspace V so that..." ?
 
Yes
 
@TedShifrin the closure of B = B union B' ( the limit Points )
kinda of an open set and then we closed it
 
OK, @Kasmir. Good. Use that to think about your question.
 
Yeah, I dunno. Seems interesting but too hard for me to say anything useful immediately
 
@TedShifrin thanks Ted :D
 
9:43 PM
And then that's just $-k(\Delta \vec{x})_A-k(\Delta \vec{x})_B-k(\Delta \vec{x})_C$
 
ill try to solve em now and come bacl later :)
 
OK.
 
@MikeMiller my problem is that I can't see any way of moving between them at all. In fact I can't even show that orthogonal projection is a locally about $p$ a homeomorphism, which I feel it must be.
@TedShifrin rewrote the question following your remark.

Given a locally Euclidean (locally homeomorphic to some Euclidean space) topological subspace $X\subset\mathbb R^n$ and $p\in X$, let $\mathrm{T}_pX$ denote the *tangent set* of $X$ at $p$, namely the set of derivatives of curves in $X$ based at $p$ and differentiable at zero.

Are the following conditions equivalent?

1. $\mathrm{T}_pX$ is a linear subspace of $\mathbb R^n$ of dimension $\dim_pX$.
2. There's a linear subspace $V\subset\mathbb R^n$ for which we have the following equality, where the limit is taken in a translated neig
 
This is also my problem. :)
 
Aww :(
Hello @BalarkaSen
 
9:47 PM
In my usual way of thinking, the inverse/implicit function theorem would tell you that you can locally represent $X$ as a graph over the tangent space and then Taylor's theorem gives your limit.
 
I can't sleep.
Hi @Arrow
 
As usual eh
What happens when you lie in bed? You're physically tired, but the sweet release of... sleep never comes?
 
metaphorically hits Balarka over the head to knock him asleep
 
@MikeMiller I like the parsing of that sentence
I am never physically tired, is the key point
;)
 
Just spiritually?
 
9:51 PM
tbh the only time I find it easy to fall asleep is when I've just woken up
 
@TedShifrin I am not able to fill out your proof sketch. Could you help me some more? I don't see any $C^1$ (or locally differentiable) function on which to apply the implicit function theorem to locally represent $X$ as a graph. I also don't understand the bit about Taylor's theorem giving the limit.
 
@TedShifrin idk if Balarka should sleep. He might have a metaphorical concussion.
 
Well, if I get to assume I start with a $C^1$ manifold, then the inverse function theorem tells me that the manifold is locally a graph over its tangent space. Taking the tangent space to be the $x_1\dots x_k$-plane, then you write the manifold as $y=f(x_1,\dots,x_k)$ ($y\in\Bbb R^{n-k}$) and the first derivatives of $f$ all vanish at the origin.
@EricSilva: Granted.
 
I am not assuming anything here except the ambient Euclidean space is a $C^1$ manifold. Moreover I want to avoid the $C^1$ assumption, but there's still the everywhere differentiable inverse function theorem. At any rate, I don't think I understand.
 
So I'm thinking your second criterion doesn't imply the first. For example, you could take $V$ to be all of $\Bbb R^n$. What happens if you start with a cone and work at the cone point?
 
9:58 PM
I don't think the tangent set is a vector subspace
At the cone point
 
In this case how is the second condition satisfied? There's no vector space which is tangent
 
It should just be... a cone again
@TedShifrin The multivariable content of the question is, suppose $f : \Bbb R^n \to \Bbb R^m$ is a function such that $Df_v(0)$ exists for all $v$. Moreover assume $\{(v, D_vf(0)) \in \Bbb R^n \times \Bbb R^m\}$ form a vector subspace of the same dimension as the graph of $f$ (which is $n$)
Is $f$ differentiable at the origin?
 
Answer: NO.
 
@BalarkaSen could you please explain why this is the heart of the matter?
 
What's an example?
@Arrow I have repeated this multiple times and am tired of doing it again
 
10:02 PM
You've done examples. All directional derivatives 0 but function not differentiable.
 
@Ted $f$ is continuous, I mean.
 
Understood. @TedShifrin so the conditions of my question are not equivalent?
 
Can you have all directional derivatives zero staying continuous?
I do not think so
 
Sure I can, @Balarka.
 
Hm, maybe take $f(x, x^2) = x$ and $f = 0$ otherwise and bump it up along the parabola?
Idk
I'd like to see a formula
 
10:07 PM
Let me ponder a moment.
 
@TedShifrin I don't understand the definition of a perfect subset
 
Why not?
 
i mean if E is closed , dont that mean every point of E is a limit point of E no ?
 
No...
You can give me a non-empty closed set with NO limit points.
 
@Ted OK, maybe you can bump the parabola up by decreasing the compact support closer and closer to the origin
That should work
 
10:11 PM
hmm
 
I fee like that should constitute a counterexample, @Arrow
 
@BalarkaSen thank you. Unfortunately I don't understand what is meant by bumping up the parabola by decreasing the compact support.
 
@TedShifrin could it be a set with bounch of Points far from each other?
 
Sure. What's the easiest example you know?
 
If it has to do with the graph of $f(x, x^2) = x$ and $f=0$ otherwise, it seems to not be locally Euclidean about the origin. (Sorry if this is completely unrelated.)
 
10:14 PM
hmm i would guess one point
 
What about an infinite set?
 
well ><
that is the thing
 
What infinite set did you know when you were 8 years old? :)
 
you told me that is can be viewd as open and closed ( that part comfused me being open and closed )
Natural numbers?
 
@Arrow The graph I have in mind is a modification of that. Multiply by a bump function $\rho$ near the parabola such that $\rho(x) \to 0$ as $x \to 0$.
 
10:15 PM
No, a point is not an open set.
Yes, natural numbers. Or $\Bbb Z$. They are closed sets with no limit points.
 
hmm your first comment what does it mean
a point is not an open set
it should be closed to make my point on that statement
 
The set consisting of a single point in $\Bbb R$ is a closed set but definitely NOT an open set.
Yes, it is a closed set, of course.
 
hmm yes i was trying to find a closed set that is not perfect
so you gave me N and Z
 
Right.
As far from perfect as you can be.
 
I need to make the definitions straight in my head :D
so Ted, the idea here, is to make the terms precise right?
what one would mean by open and closed and limit point ect
 
10:20 PM
Well, sure, that's the beginning.
 
so we get a common ground so to speak :D
 
@BalarkaSen sorry for frustrating you, I'm just not able to follow your descriptions. Do you want the bump function to "flatten" the tilted parabola near the origin, thereby making the tangent set the $xy$-plane and yet leaving the resulting subset without a tangent plane at the origin in the sense of the second condition?
 
ehm nice
btw Ted , a notation here that is not very clear
in Rudin {E_alpha}
and sometimes Union_alpha E_alpha
from what I understood , its a set of sets
but the point here, the element of E_alpha comes from what set?
and the family of sets, do we consider the sets as elements or what inside those sets?
 
@Balarka: Well, I can go back to your original statement. I can easily give you a non-differentiable function that is continuous and has directional derivatives in every direction at 0.
Each $E_\alpha$ is a subset of your master space $X$.
This is a confusing point in set theory. You have a collection of subsets of $X$.
 
@TedShifrin I think $f(x,y)=\frac{x^3y}{x^4+y^2}$ works for this? (The graph is even locally Euclidean.)
 
10:25 PM
That probably ends up being differentiable at $0$, though?
 
No, because of $f(x,x^2)=\frac x2$ - this parabola has a tangent not within the $xy$-plane.
 
Here's an easy one. $f(x,y) = |x|y/\sqrt{x^2+y^2}$. Continuous. Has all directional derivatives, but no tangent plane.
Oh, good, @Arrow.
 
Oh, I guess my reformulation of Arrow's question is flawed
 
My issue with @Arrow's second condition is that I could just take $V$ to be the ambient vector space, so the projection on $V^\perp$ is null.
 
Ah, so restrict $V$ to also have the right dimension.
 
10:30 PM
I'm not really sure what it means for $V$ to have the right dimension
 
$\dim V=\dim _pX$ since we're assuming $X$ is locally Euclidean.
 
Me either.
 
Oh, I didn't know we were asusming that of X
 
So that would rule out taking the Zariski tangent space of larger dimension.
 
Yes, I wrote it in the question I think.
 
10:31 PM
My bad
 
You did, @Arrow.
 
Without this assumption we could take a paraboloid and an upside down paraboloid "tangent" at the origin: their union would satisfy the second condition but not the first I think.
 
The right question is probably this. Suppose $f : \Bbb R^n \to \Bbb R^m$ is a continuous function such that for any curve $\alpha$ going through the origin, $(0, (f \circ \alpha)'(0))$ exists and they form a vector space of dimension $n$ inside $\Bbb R^{m+n}$. Does that mean $f$ is differentiable at $0$?
Right, I was just thinking about $\alpha$ to be linear directions
That is not what his question is
 
If I understand correctly, this might answer affirmatively?
 
I think it's Ted's question about if $f$ is continuous along every path through the origin then $f$ is continuous
Just ramped up in smoothness order
 
10:35 PM
Theorem 3.3.2 in maths.ed.ac.uk/sites/default/files/atoms/files/khwaja.pdf says "$|ph(1-z)|<\pi$ and $|ph(z+1)|<\pi$". Is it meant that only one of these conditions is enough? I'm having this doubt because taking $z < -1$ (which satisfies only the first condition) also happens to work. $ph$ is complex argument or phase
 
@Arrow Right, there you go. It's the same solution
Find a sequence of points along which $f$ is not differentiable, then interpolate them by a smooth curve
plus stuff near the origin
 
Glad to know I've created some memories in your brain, Balarka :P
 
@BalarkaSen I just don't understand how this proves the equivalence between my conditions, where seemingly no functions apart from the inclusions are involved.

Regarding what you propose - the problem with this interpolation is that I see no way of making the resulting curve actually within $X$.
 
Use the ball in Euclidean space that your $X$ is locally homeomorphic to?
Not that I'm really following ... There's a ton of ruckus going on with repairmen in my house.
 
Using a homeomorphism might not preserve the crucial differentiability of the interpolation curve at zero.
If indeed I could interpolate such a sequence to get a curve taking values in $X$ which is differentiable at zero, this seems to prove the first condition implies the second.
@TedShifrin I am desperate here :) Does your intuition tell you these condition are equivalent or not, by the way?
 
10:41 PM
So how do you even define the tangent space if I only have a homeomorphism that isn't differentiable? How do I define differentiable curves?
 
We're in a Euclidean space, so just use that.
 
It sits inside R^n
 
Oh, right.
 
This is annoying
 
I asked the question here by the way, if anyone cares.
 
10:45 PM
So you'd have to find a topological submanifold of R^n and a point p there such that no neighborhood of p is locally differentiably homeomorphic (with inverse also differentiable) to R^k. I think the bad points of Alexander's horned sphere is a good starting point :P
It has to be really bad I mean
 
I don't think any bad point of that thing satisfies either of the conditions though
 
I don't see it well enough to be convinced about anything
 
If you don't then certainly I don't :)
 
It could be potentially true that no curve passing through the bad point of Alexander's horned sphere is differentiable at the bad point as a curve in R^3
@Mike, @Ted, is this plausible?
 
God I have no idea
I work with simple things so I don't need to think so hard, haha
 
10:49 PM
lol fair
If @Akiva doesn't know probably none of us knows
 
I only think (thought) about singularities in the complex analytic/algebraic category.
LOL ... DogAteMy the guru.
 
the guru of infinitely botched things
that could be a good title
 
Digressing, does anyone see how to prove the second condition implies $\pi_V$ is locally a homeomorphism about $p$? The locally Euclidean hypothesis will be needed otherwise a pair of "kissing" paraboloids will be a counterexample.
 
I can't imagine there being anything differentiable about the horned sphere.
 
Yeah get him
I think the reason you're having trouble finding people discussing this in the literature might be that it's 3hard 5them
 
10:51 PM
$\pi_{V^\perp}$ gives you the representation of the object as a graph of (presumably) a function over $V$.
Rob Kirby would know the answer.
Or at least would have 40 years ago.
 
11
Q: Perturbation of a smooth manifold and transversaility

user119986Let $M$ be a compact smooth manifold and $N$ be a compact smooth submanifold of $M$. The usual transversality theorem claims that for a generic diffeomorphism $f$ of $M$, the submanifolds $N$ and $f(N)$ are transverse. I am interesting in another point of view. The diffeomorphism $f$ is now fi...

 
Hello @PVAL
 
This question looked interesting
I suspect its true if the graph of f is transverse to the diagonal.
I couldn't think up a proof though.
 
That's kinda cool
 
Hmm, also too hard for me.
 

« first day (2730 days earlier)      last day (2308 days later) »